大学物理课后习题答案详解

第一章质点运动学

1、(习题1.1):一质点在xOy平面内运动,运动函数为x=2t,y=4t?8。(1)求质点的轨道方程;(2)求t=1 s和t=2 s 时质点的位置、速度和加速度。 解:(1)由x=2t得,

22

y=4t-8 可得: y=x-8 即轨道曲线 (2)质点的位置 : r?2ti?(4t?8)j 由v?dr/dt则速度: v?2i?8tj 由a?dv/dt则加速度: a?8j

则当t=1s时,有 r?2i?4j,v?2i?8j,a?8j 当t=2s时,有 r22?4i?8j,v?2i?16j,a?8j

2、(习题1.2): 质点沿x在轴正向运动,加速度a??kv,k为常数.设从原点出发时速

度为v0,求运动方程x?x(t).

解:

dv??kv dtdx?v0e?kt dtt1?ktv?ve dv??kdt0?v0v?0v

?x0dx??v0e0t?ktdt x?v0(1?e?kt) k3、一质点沿x轴运动,其加速度为a ? 4t (SI),已知t ? 0时,质点位于x ??10 m处,初速度v??? 0.试求其位置和时间的关系式. 解: a?dv /dt?4t dv ?4t dt v?dx /d t?2t2

?v0dv??4tdt v?2t2

0t?xx0dx??2t2dt x?2 t3 /3+10 (SI)

0t4、一质量为m的小球在高度h处以初速度v0水平抛出,求:

(1)小球的运动方程;

(2)小球在落地之前的轨迹方程; (3)落地前瞬时小球的

drdvdv,,. dtdtdt解:(1) x?v0t 式(1)

11y?h?gt2 式(2) r(t)?v0ti?(h-gt2)j

22gx2(2)联立式(1)、式(2)得 y?h?22v0 (3)

dr?v0i-gtj 而落地所用时间 t?dtdr2h 所以 ?v0i-dtg 2ghjg2ghdvdvg2t2222 ????gj v?vx?vy?v0?(?gt) 11222dt[v?(gt)]2(v?2gh)2dt005、 已知质点位矢随时间变化的函数形式为r?ti?2tj,式中r的单位为m,t的单位为s.求:(1)任一时刻的速度和加速度;(2)任一时刻的切向加速度和法向加速度。

解:1)v?2drdv?2ti?2j a??2i dtdt12 2)v?[(2t)2?4] at??2(t2?1)2

an?1dv?dt2tt?12a2?at2?2t?12

第二章质点动力学

1、(牛顿定律)质量为M的气球以加速度a匀加速上升,突然一只质量为m的小鸟飞到气球上,并停留在气球上。若气球仍能向上加速,求气球的加速度减少了多少? 解:f为空气对气球的浮力,取向上为正。 分别由图(a)、(b)可得:

F?Mg?Ma

F?(M?m)g?(M?m)a1

则a1?

2、 (牛顿定律) 两个圆锥摆,悬挂点在同一高度,具有不同的悬线长度,若使它们运动时两个摆球离开地板的高度相同,试证这两个摆的周期相等.

证:设两个摆的摆线长度分别为l1和l2,摆线与竖直轴之间的夹角分别为?1和?2,摆线中的张力分别为F1和F2,则

F1cos?1?m1g?0 ① F1sin?1?m1v1/(l1sin?1) ② 解得:

2Ma?mgm(a?g) ,a?a?a1?m?Mm?M v1?sin?1gl1/cos?12?l1sin?1?2?v1 第一只摆的周期为 m1 T1?m2 l1cos?1

g同理可得第二只摆的周期 T2?2?l2cos?2 g由已知条件知 l1cos?1?l2cos?2 ∴ T1?T2 习题2.1—2.6

习题2.1一颗子弹在枪筒里前进时所受的合力大小为F?400?4?105t/3,子弹从枪口射出时的速率为300m/s。设子弹离开枪口处合力刚好为零。求:(1)子弹走完枪筒全长所用的时间t;(2)子弹在枪筒中所受力的冲量I;(3)子弹的质量。

解:(1)由F?400?4?105t/3和子弹离开枪口处合力刚好为零,则可以得到:F?400?4?105t/3?0 算出t=0.003s。

(2)由冲量定义:

I??Fdt??(400?4?105t/3)dt?400t?2?105t2/3003330?0.6N?s

(3)由动量定理: I?3Fdt??P?mv?0.6N?s?0 所以:m?0.6/300?0.002kg 习题2.2 质量为M=1.5 kg的物体,用一根长为l=1.25 m的细绳悬挂在天花板上.今有一质量为m=10 g的子弹以v0=500 m/s的水平速度射穿物体,刚穿出物体时子弹的速度大小v =30 m/s,设穿透时间极短.求:

?v0 m l ?v M (1) 子弹刚穿出时绳中张力的大小; 习题 2.2 图 (2) 子弹在穿透过程中所受的冲量.

解:(1)取子弹与物体为研究对象,子弹前进方向为x轴正向, 因穿透时间极短,故可认为物体未离开平衡位置.因此,作用于子弹、物体系统上的外力均在竖直方向,故系统在水平方向动量守恒.令子弹穿出时物体的水平速度为v? 有 mv0 = mv+M v?

v? = m(v0 ? v)/M =3.13 m/s

T =Mg+Mv2/l =26.5 N

(2) f?t?mv?mv0??4.7N?s (设v0方向为正方向) 负号表示冲量方向与v0方向相反.

习题2.3一人从10 m深的井中提水.起始时桶中装有10 kg的水,桶的质量为1 kg,由于水桶漏水,每升高1 m要漏去0.2 kg的水.求水桶匀速地从井中提到井口,人所作的功.

解:选竖直向上为坐标y轴的正方向,井中水面处为原点. 由题意知,人匀速提水,所以人所用的拉力F等于水桶的重量 即: F?P?P0?ky?mg?0.2gy?107.8?1.96y 人的拉力所作的功为: W??dW??Fdy=?(107.8?1.96y)dy=980 J

00H10习题2.4 如图所示,质量m为 0.1 kg的木块,在一 个水平面上和一个劲度系数k为20 N/m的轻弹簧碰撞,木块将弹簧由原长压缩了x = 0.4 m.假设木块与水平面间的滑动摩擦系数? 为0.25,问在将要发生碰撞时木块的速率v为多少?

解:根据功能原理,木块在水平面上运动时,摩擦力所作的功等于系统(木块和弹簧)机械能的增量.由题意有 ?frx?而

121kx?mv2 22m 习题2.4图

k fr??kmg

木块开始碰撞弹簧时的速率为

v?kx22?kgx??5.83ms

m习题2.5某弹簧不遵守胡克定律. 设施力F,相应伸长为x,力与伸长的

关系为 F=52.8x+38.4x2(SI)求:

(1)将弹簧从伸长x1=0.50 m拉伸到伸长x2=1.00 m时,外力所需做的功.

(2)将弹簧横放在水平光滑桌面上,一端固定,另一端系一个质量为2.17 kg的物体,然后将弹簧拉伸到一定伸长x2=1.00 m,再将物体由静止释放,求当弹簧回到x1=0.50 m时,物体的速率.

解:(1) 外力做的功

x1x112mv??F'?dx???Fdx?W?31Jx2x22(2) 设弹力为F′

v?2Wm?5.34ms?1习题2.6两个质量分别为m1和m2的木块A、B,用一劲度系数为k的轻弹簧连接,放在光滑的水平面上。A紧靠墙。今用力推B块,使弹簧压缩x0然后释放。(已知m1?m,m2?3m)求:(1)释放后A、B两滑块速度相等时的瞬时速度的大小;(2)弹簧的最大伸长量。

解:

1122m2v20?kx0 22m2v20?(m1?m2)v 所以v?习题2.6图

3kx0 43m11112(2)m2v20?kx2?(m1?m2)v2 计算可得:x?x0

22223、(变力作功、功率、质点的动能定理)设F?7i?6j(N)(1)当一质点从原点运动到r??3i?4j?16k(m)时,求F所作的功;(2)如果质点到r处时需0.6s,试求F的平

均功率;(3)如果质点的质量为1kg,试求动能的变化。 解:(1)A=rr-34?0F?dr=?(7i?6j)?(dxi?dyj?dzk)=?7dx??6dy??45J,做负功

000r4A45 (2)P??-45+??mgdy = -85J ?75W (3)?Ek?A???mgj?dr =

00t0.64、(机械能守恒、动量守恒)如图所示,一个固定的光滑斜面,倾角为θ,有一个质量为m小物体,从高H处沿斜面自由下滑,滑到斜面底C点之后,继续沿水平面平稳地滑行。设m所滑过的路程全是光滑无摩擦的,试求:(1)m到达C点瞬间的速度;(2)m离开C点的速度;(3)m在C点的动量损失。 解:(1)由机械能守恒有 mgH?12mvc 2带入数据得vc?2gH,方向沿AC方向 (2)由于物体在水平方向上动量守恒,所以

mvccos??mv,得v?2gHcos?,方向沿CD方向

(3)由于受到竖直的冲力作用,m在C点损失的动量?p?m2gHsin?,方向竖直向下。

第三章刚体的运动

书:3.3用落体观察法测定飞轮的转动惯量,是将半径为R的飞轮支承在O点上,然后在绕过飞轮的绳子的一端挂一质量为m的重物,令重物以初速度为零下落,带动飞轮转动,记下重物下落的距离和时间,就可算出飞轮的转动惯量。试写出它的计算式。(假设轴承间无摩擦

解:如习题3.3(b)图,对飞轮而言,根据转动定律,

FTR?J?O

FT'm mg FT(1)

对重物而言,由牛顿定律,有

mg?FT'?ma F'T?FT (2)

由于绳子不可伸长,因此,有

习题3.3(b)图

a?R? (3)

重物作匀加速下落,则有

h?12at (4) 2gt2?1) 由上述各式可解得飞轮的转动惯量为 J?mR(2h23.4如图,一轻绳跨过两个质量为m、半径为r的均匀圆盘状定滑轮,绳的两端分别挂着质量为2m和m的重物,绳与滑轮间无相对滑动,滑轮轴光滑,两个定滑轮的转动惯量均为mr2/2,将由两个定滑轮以及质量为2m和m的重物组成的系统从静止释放,求重物的加速度和两滑轮之间绳内的张力。

解:受力分析如图

2mg?T2?2ma (1) T1?mg?ma (2)

(T2?T)r?J? (3) (T?T1)r?J? (4)

习题3.4图

a?r? (5)

联立 a?111g, T?mg 483.6有一质量为m1、长为l的均匀细棒,静止平放在滑动摩擦系数为?的水平

桌面上,它可绕通过其端点O且与桌面垂直的固定光滑轴转动。另有一水平运动的质量为m2的小滑块,从侧面垂直于棒与棒的另一端A相碰撞,设碰撞时间极短。已知小滑块在碰撞前后的速度分别为v1和v2,如图所示。求碰撞后从细棒开始转动到停止转动的过程所需的时间。

1 (已知棒绕O点的转动惯量J?m1l2)

3 解:碰撞时角动量守恒

1m2v1l?m1l2w?m2v2l

3??3m2(v1?v2)

m1l细棒运动起来所受到的摩擦力矩

M???0lm11gxdx??m1gl l22? ?Mdt?J?0t?J?1?0?J?习题3.6图

1

m1l2?t?3 1

?m1gl2

t?2l?2m2(v1?v2)? 3?g?m1g

1. 如图所示,物体1和2的质量分别为m1与m2,滑轮的转动惯量为J,半径为r, 物体2与桌面间的摩擦系数为?,设绳子与滑轮间无相对滑动,滑轮与转轴无摩擦。求系统的加速度a 及绳中的张力T1和T2。 m1g?T1?m1a

2T2??m2g?m2a T1r?T2r?J? a?r?

1m1?m1??m2?gr2T1?m1g?解得:a?

J?m1r2?m2r2 J?m1r2?m2r2

2、如图系统中,m1=50kg, m2=40kg,圆盘形滑轮m=16kg,半径r=0.1m,斜面是光滑的,倾角θ=300,绳与滑轮无相对滑动,转轴摩擦不计,求:

(1)绳中的张力;(2)设开始时m1距离地面高度为1m,需多长时间m1到达地面?

?m1??m2?gr2m1g?T1?m1a T2?m2gsin??m2a

T1r?T2r?J?

a?r?

J?1222mr解得 ??30rad/s,a?3m/s ,T1?340N,T2?316N 2

12由h?v0t?at,v0?0,所以t?2

2h?0.816s a3.一长为1 m的均匀直棒可绕过其一端且与棒垂直的水平光滑固定轴转动.抬起另一端使

12ml3棒向上与水平面成30°,然后无初转速地将棒释放.已知棒对轴的转动惯量为,求:

(1) 放手时棒的角加速度; (2) 棒转到水平位置时的角速度.

解: 1、??M JM?mgl31cos300?mgl J?ml2 2433mgl33g33g??4??

124l4ml32、机械能守恒

l1mgsin300mgl124?3g?2??mgsin300?0?0?J?21121222 ?mlml236

??3g=3.83rad/s 24.一根长为l、质量为M的匀质棒自由悬挂于通过其上端的光滑水平轴上。现有一质量为m的子弹以水平速度v0射向棒的中心,并以v0/2

lv 0 0 的水平速度穿出棒,此后棒的最大偏转角恰为90?,求v0的大小。 mmMv/2 角动量守恒 mv01lmv0l??J? J?Ml2 2223ll14?3mv0 mv0?Ml2? ??124Ml43Ml3mv0112?3mv0?l1122l机械能守恒?Ml??Mg ?Ml???Mg

232232?4Ml? v0224M16M2lv??g 0m3m2gl 35.一根长为l、质量为 M的匀质棒自由悬挂于通过其上端的光滑水平轴上。现有一质量为 m?1M的子弹以水平速度v0射入棒的下端,并留在棒里。此后棒的最大偏转角恰为6角动量守恒

60°,求v0。

v11mv0l?(ml2?Ml2)?m?M??0

3l 36

机械能守恒

11l(ml2?Ml2)?2?Mg(1?cos600)?mgl1?cos600232??

v0?23gl

6、如图所示,长为l的轻杆,两端各固定质量分别为m和2m的小球,杆可绕水平光滑固定轴O在竖直面内转动,转轴O距两端分别为l和

132l.轻杆原来静3止在竖直位置。今有一质量为m的小球,以水平速度v0与杆下端小球

1m作对心碰撞,碰后以v0的速度返回,试求碰撞后轻杆所获得的角

2速度。

解:角动量守衡

22ll21mv0l?()2m??()2?2m??ml?v0 33332??3v0 2l

第四章振动与波动

振动部分:习题4.2、4.4、4.5

习题4.2一物体沿x轴做简谐运动,振幅为0.06 m,周期为2.0 s,当t = 0时位移为0.03m,且向x轴正方向运动。求:(1)t = 0.5 s时,物体的位移、速度和加速度;(2)物体从x = ?0.03m 处向x轴负向运动开始,到平衡位置,至少需要多少时间?

解: (1)由题意知A = 0.06m、??2?T??s?1由旋转矢量(a)图可确定初相则?0???3,振动方程为

?1 x?(0.06m)cos?(?s)t??3???

习题4.2 (b) 图

习题4.2 (a) 图

当t = 0.5s时质点的位移、速度、加速度分别为

x?(0.06m)cos(?2??3)?0.052mv?dxdt??(0.06?m?s?1)sin(?2??3)??0.094m?s?1a?d2xdt2??(0.06?2m?s?2)cos(?2??3)??0.513m?s?2

(2)质点从x =?0.03 m运动到平衡位置的过程中,旋转矢量从(b)图中的位置M转至位置N,矢量转过的角度(即相位差)???5?6。该过程所需时间为

?t?????0.833s

习题4.4 某质点振动的x-t曲线如题图所示.求:(1)质点的振动方程; (2)质点到达P点相应位置所需的最短时间.

解:(1)设所求方程为:x=Acos(ωt+φ)0从图中可见,t=0,x0=A/2,v0>0由旋转矢量法可知;φ0=-又t=1s,ωt-5π65πt-π)mπ3=π2π3?ω=

习题4.4图

63(2)P点的相位为0??t??0?5?故:x=0.1cos(t??0t?0.4spp6p3即质点到达P点相应状态所要的最短时间为0.4s?习题4.5一质点沿x轴作简谐振动,振幅为12cm,周期为2s。当t?0时, 位移为6cm,且向x轴正方向运动。求:(1)振动表达式;(2)t?0.5s时,质点的位置、速度和加速度;(3)如果在某时刻质点位于x??6cm,且向x轴负方向运动,求从该位置回到平衡位置所需要的时间。

解:由题已知 A=12×10-2m,T=2.0 s ∴ ω=2π/T=πrad·s-1

?又,t=0时,x0?6cm,v0?0 ∴由旋转矢量图,可知:?0??

3(?t?)故振动方程为x?0.12cos

3 (2)将t=0.5 s代入得

?x?0.12cos(?t?)?0.12cos?0.103m

36v??0.12?sin(?t?)?0.12cos??0.189m/s 36a??0.12?2cos(?t?)??0.12?2cos??1.03m/s2 36??????方向指向坐标原点,即沿x轴负向.

(3)由题知,某时刻质点位于x??6cm,且向x轴负方向运动

即x0=-A/2,且v<0,故?t=2π/3,它回到平衡位置需要走5π/6,所以: ∴t=Δ?/ω=(5π/6)/(π) =5/6s

习题4.5图

(加题)1.有两个同方向同频率的振动,其合振动的振幅为0.2m,合振动的相位与第一个振动的相位差为?/6,第一个振动的振幅为0.173m,求第二个振动的振幅及两振动的相位差。

分析 根据已知振幅和相位可在矢量三角形中求得振幅。 解:采用旋转矢量合成图求解

取第一个振动的初相位为零,则合振动的相位为???/6 据A?A1?A2可知A2?A?A1,如图:

A2?A1?A2?2AA1cos??0.1(m)

由于A、A1、A2的量值恰好满足勾股定理, 故A1与A2垂直.

即第二振动与第一振动的相位差为???/2

(加题)2.一质点同时参与两个同方向的简谐振动,其振动方程分别为

题图5-26

2x1?5?10?2cos(4t??/3)(SI),x2?3?10?2sin(4t??/6)(SI)画出两振动的旋转矢量图,

并求合振动的振动方程.

分析 须将方程转化为标准方程从而确定其特征矢量,画出矢量图。 解:x2?3?10?2sin(4t??/6)

?3?10?2cos(4t??/6??/2) cos(4t?2?/3)

?3?10?2作两振动的旋转矢量图,如图所示. 由图得:合振动的振幅和初相分别为

A?(5?3)cm?2cm,???/3.

合振动方程为x?2?10?2题图5-27

cos(4t??/3)(SI)

(加题)3.一物体质量为0.25kg,在弹性力作用下作简谐振动,弹簧的劲度系数

J和动能0.02 J,求 (1) 振幅;(2) 动k?25N?m?1,如果起始振动时具有势能0.06

能恰等于势能时的位移;(3) 经过平衡位置时物体的速度. 解:(1) E?EK?Ep? (2)

12kA=0.08 A?22?0.08?0.08m 25121kx?mv2 ; k?m?2 ?m?2x2?m?2A2sin2(?t??) 222222222 x?Asin(?t??)?A[1?cos(?t??)]?A?x

2x2?A2,?x??A/2??0.0566m

(3) 过平衡点时,x?0,此时动能等于总能量

E?EK?Ep?1mv2=0.08 A?22?0.08?0.8m/s

0.25(加题)4. 一弹簧振子,弹簧的劲度系数为k=25N/m,当物体以初动能0.2J和初势能0.6J振动时,求: (1) 振幅是多大? (2) 位移多大时,其势能和动能相等? (3) 位移是振幅的一半时,势能是多大?

解: (1) 弹簧振子的总机械能为E?Ek?Ep?2(Ek?Ep)12kA,故A??0.253m 2k (2) Ep?Ek?21111A??0.179m E?kA2 kx2?kA2 x??22424121A2?0.20J (3) Ep?kx?k224波动部分:习题4.7、4.8、4.10

习题4.7有一平面简谐波在介质中传播,波速u = 100 m/s,波线上右侧距波源O(坐标原点)为75.0 m处的一点P的运动方程为

习题4.7图

yp?(0.30m)cos[(2?s?1)t??/2]。求(1)波向x轴正方向传播时的波动方程;(2)

波向x轴负方向传播时的波动方程。

解:(1)设以波源为原点O,沿x轴正向传播的波动方程为 y?Acos???t?xu???0?

将 u = 100 m?s?1代人,且取x = 75 m得点P的运动方程为 yP?Acos???t?0.75s???0?

与题意中点P的运动方程比较可得 A = 0.30 m、??2?s?1、?0?2?。则所求波动方程为

y?(0.30m)cos[(2?s?1)(t?x/100m?s?1)]

(2)当沿x轴负向传播时,波动方程为 y?Acos???t?xu???0?

将 x = 75 m、u?100ms?1代人后,与题给点P的运动方程比较得A = 0.30 m、??2?s?1、?0???,则所求波动方程为

?1?1 y?(0.30m)cos[(2?s)(t?x/100m?s)??]

讨论:对于平面简谐波来说,如果已知波线上一点的运动方程,求另外一点的运动方程,也可用下述方法来处理:波的传播是振动状态的传播,波线上各点(包括原点)都是重复波源质点的振动状态,只是初相位不同而已。在已知某点

???0?2??x/?初相?0的前提下,根据两点间的相位差????0,即可确定未知点的

?初相?0。

1习题4.8已知一沿x正方向传播的平面余弦波,t?s时的波形如题图所示,

3且周期T为2s.

(1)写出O点的振动表达式; (2)写出该波的波动表达式; (3)写出A点的振动表达式; (4)写出A点离O点的距离。

习题4.8图

解:由图可知A=0.1m,λ=0.4m,由题知T= 2s,ω=2π/T=π,而u=λ/T=0.2m/s。 波动方程为:y=0.1cos[π(t-x/0.2)+Ф0]m 关键在于确定O点的初始相位。 (1) 由上式可知:O点的相位也可写成:φ=πt+Ф0

1由图形可知: t?s时y0=-A/2,v0<0,∴此时的φ=2π/3,

32?1?将此条件代入,所以:????0 所以?0?

333O点的振动表达式y=0.1cos[πt+π/3]m

(2)波动方程为:y=0.1cos[π(t-x/0.2)+π/3]m (3)A点的振动表达式确定方法与O点相似由上式可知:

A点的相位也可写成:φ=πt+ФA0

1由图形可知: t?s时y0=0,v0>0,∴此时的φ=-π/2,

3?15?将此条件代入,所以:?????A0 所以?A0??

236A点的振动表达式y=0.1cos[πt-5π/6]m

(4)将A点的坐标代入波动方程,可得到A的振动方程,与(3)结果相同,所以: y=0.1cos[π(t-x/0.2)+π/3]= 0.1cos[πt-5π/6]

可得到:xA?7?0.233m 30习题4.10 一平面简谐波以速度u?0.8m/s沿x轴负方向传播。已知原点的振动曲线如图所示。试写出:

(1)原点的振动表达式; (2)波动表达式;

(3)同一时刻相距1m的两点之间的位相差。

解:(1) 由图可知A=0.5cm,原点处的振动方程为:y=Acos(ωt+φ) t=0s时 y=A/2 v>0 可知其相位为φ=?习题4.10图

? 3 t=1s时 y=0 v<0 可知其相位为φ1=

? 2 代入振动方程, φ=??? ω+φ= 32可得:ω=

5? T=2π/ω=12/5 65??t-)cm 635?5x?(t+)-]cm 643 则 y=0.5cos(

(2)沿x轴负方向传播,波动表达式:y=0.5cos[(3)根据已知的T=12/5,u?0.8m/s,可知:??48m 25?x25那么同一时刻相距1m的两点之间的位相差:???2????3.27rad

?24

(加题)1.如图,一平面波在介质中以波速u?20m/s沿x轴负方向传播,已知A点的振动方程为y?3?10?2cos4?t(SI).

u

B A 1题图

(1)以A点为坐标原点写出波方程;

(2)以距A点5m处的B点为坐标原点,写出波方程. 解:(1)坐标为x处质点的振动相位为

?t???4?[t?(x/u)]?4?[t?(x/20)] 波的表达式为 y?3?10?2cos4?[t?(x/20)](SI)

(2)以B点为坐标原点,则坐标为x点的振动相位为 ?t??'?4?[t?波的表达式为 y?3?10?2x?5](SI) 20x?5??cos4??t??

20?? y?3?10?2cos[4?(t?x)??](SI) 20(加题)2. 一平面谐波沿ox轴的负方向传播,波长为λ,P点处质点的振动规律如题图6-10所示.求:

(1)P点处质点的振动方程; (2)此波的波动方程;

(3)若图中d??/2,求O点处质点的振动方程.

分析 首先由已知振动规律结合旋转矢量图可得P点振动的初相与周期,从而得到其振动方程。波动方程则由P与原点的距离直接得到。波动方程中直接代入某点的坐标就可求出该点的振动方程。

解:(1)从图中可见T?4s,且t?0,ypo??A,??0??,则P点处质点的振动方程为

2??yp?Acos(t??)?Acos(t??)(SI)

42(2)向负方向传播的波动方程为

yP (m) ???4?x?d???y?Acos??t?????

???2??(3)把d??/2,0 1 -A O t (s) d P x x?0代入波动方程即得

y0?Acost

2

(加题)3.两波在一很长的弦线上传播,其波方程分别为:

?题图6-10

131 y2?4.00?10?2cos?(4x?24t)(SI)

3 y1?4.00?10?2cos?(4x?24t)(SI)

求:(1)两波的频率、波长、波速;(2)两波叠加后的波节位置;(3)叠加后振幅最大的那些点的位置.

解:(1)与波动的标准表达式y?Acos2?(?t?x/?)对比可得: ??4Hz, ??1.50m, 波速u????6.00m/s (2)波节位置4?x/3??(n??131?)即x??(n?)m,n?0,1,2... 242(3)波腹位置4?x/3??n?即x??3n/4m,n?0,1,2... 第11章作业

11.2 在双缝装置中,用一很薄的云母片(n =1.58)覆盖其中的一条狭缝,这时屏幕上的第七级明条纹恰好移到屏幕中央(原零级明条纹)的位置。如果入射光的波长为550nm,则这云母片的厚度应为多少?

分析:云母片覆盖前,零级明条纹在屏幕上O点。覆盖后,衍射条纹移动了7条,即第七条明条纹位于O点。由光程差的变化计算介质厚度。

习题11.2图

解:覆盖前,两光到达O点的光程差为

?1?r2?r1?0 (1)

覆盖后,两光到达O点的光程差为

?2??r2?e??ne?r1?7? (2)

(2)式与(1)式作差,可得

?2??1???r2?e??ne?r1???r2?r1??e?n?1??7?

所以

7?7?550?10?9e???6.64?10?6m

n?11.58?111.3 在双缝实验中,入射光是由波长?1?550nm和另一束未知波长?2两种成分合成的复色光。已知双缝间距为0.6mm,屏和缝的距离为1.2m,求屏上?1的第三级明纹中心的位置。若屏上?1的第六级明纹中心和未知的?2的第五级明纹中心重合,求未知波长?2。

分析:由明纹中心位置公式x?k解:第三级明纹中心位置

D?1.2?550?10?9?3??3.3mm x?kd0.6?10?3D?可得。 d?1的第六级明纹中心和未知的?2的第五级明纹中心重合,即它们具有相同的衍

射角

6所以

?2?66?1??550?660nm 55D?1D?2?5 dd11.5 一薄玻璃片,厚度为0.40μm,折射率为1.5,置于空气中。用白光垂直照射,问在可见光的范围内,哪些波长的光在反射中加强?哪些波长的光在透射中加强?

分析:分别应用反射光和透射光在等倾干涉中加强的条件求得。 解:反射加强的条件为 2ne??2?k?

由此得

??4ne 2k?14?1.50?0.40?480nm

2?3?1仅当k?3时,?为可见光,因此求得

??透射加强的条件即反射减弱的条件,即

2ne????2k?1? 22?由此得

??当k?2时,

??当k?3时,

??4?1.50?0.40?400nm

2?34?1.50?0.40?600nm

2?24ne 2k 波长为480nm的可见光在反射中加强,波长为600nm和400nm的可见光在透射中加强。

11.6 一单色光垂直照射在厚度均匀的薄油膜上,油膜覆盖在玻璃板上。油的折射率为1.3,玻璃的折射率为1.5,若单色光的波长可由光源连续可调,并观察到500nm与700nm这两个波长的单色光在反射中消失,求油膜的厚度。

分析:由于玻璃的折射率n2大于油的折射率n1,光线在油膜上,下表面反

00nm的光在k级干涉相消,射时都存在半波损失,则光程差为??2ne。设?1?5则对于?2?700nm的光在第?k?1?级干涉相消。

解: 对?1?500nm的光在k级干涉相消,有 2n1e??2k?1??11????k???1 (1) 2?2?对?2?700nm的光在第?k?1?级干涉相消,有

2n1e??2?k?1??1??21????k???2 (2) 2?2?由(1)、(2)式解得

k??1??2500?700??3

2??2??1?2?700?500?1???3???5002???673.1nm 2?1.301???k???12? e???2n111.7 有一玻璃劈尖,玻璃的折射率为1.5,劈尖夹角??5?10?5rad。用单色光垂直照射,测得相邻两条明纹间的距离l?3.64?10?3m,求此单色光的波长。

分析:由相邻两明纹间的距离公式可得。 解:相邻两明纹的距离为 l??2nsin?,

因为?很小,所以sin???,则

l?所以,

??2nl?

=2?1.5?3.64?10?3?5?10?5 =546nm

11.11 在折射率n1?1.52的照相机镜头表面镀有一层折射率n2?1.38的MgF2增透膜,如果此膜适用于波长??550nm的光,问膜的最小厚度应是多少?

分析:由薄膜干涉公式可得。

解:对穿过增透膜的透射光而言,两相干光的光程差为2ne?波长的透射光增强,应满足条件

2ne?当k?1时,对应膜的最小厚度

2nemi?n? 2n??2,为使给定

?2?k?

?2

已知??550nm,n2?1.38,由此可以算出膜的最小厚度

e??4n2?99.4nm

11.12 (1)若用波长不同的光观察牛顿环,?1?600nm,?2?450nm,观察利用?1时的第k个暗环与用?2时的第k+1个暗环重合,已知透镜的曲率半径是190cm。求用?1时第k个暗环的半径。

(2)又如在牛顿环中用波长为500nm的第5个明环与用波长为?3时的第6个明环重合,求波长?3。

1??分析:用牛顿环暗环半径公式r?kR?,明环半径公式r??k??R?计

2??算求得。

解:(1)?1的第k个暗环半径为 rk?kR?1

?2的第k+1个暗环半径为

rk?1??k?1?R?2

两个暗环重合,即rk?rk?1,由以上式子可得k?3,代入下式

rk?kR?1?3?190?10?2?600?10?9?1.85?10?3m

1??(2)由明环半径公式r??k??R?,且波长为500nm的第5级明纹与波长?3时

2??的第6个明环重合。可得

1?1??? ?5??R???6??R?3

2?2???所以

?3?99???500?409.1nm 111111.13 当观察牛顿环装置中的透镜与玻璃板之间的空间充以某种液体时,第10个明环的直径由1.40?10?2m变为1.27?10?2m,试求这种液体的折射率。

分析:当透镜与与平板玻璃间充满某种液体(n2?1),且满足n1?n2,n2?n3 或n1?n2,n2?n3时,在厚度为e的地方,两相干光的光程差为??2n2e?此可推导出牛顿环暗环半径r??2。由

kR?1?R??和明环半径r??k??,这里明,暗

2?n2n2?环半径和充入的介质折射率n2有关。在牛顿环公式中,若介质不均匀或分析的是透射光而不是反射光,那么关于暗环,明环半径的公式与教材中的公式是不同的。

解:当透镜与玻璃之间为空气时,k级明纹的直径为

1??dk?2rk?2?k??R?

2??当透镜与玻璃之间为液体时,k级明纹的直径为

1?R?? dk'?2rk'?2?k??

2?n2?解上述两式得

?dk n2???d'?k????1.22 ?211.17 波长??500nm的平行单色光,垂直入射到宽度为a=0.25mm的单缝上,紧靠单缝后放一凸透镜,如果置于焦平面处的屏上中央明纹两侧的第三级暗纹之间的距离是3mm,求透镜焦距。

分析:由单缝衍射暗纹条件及暗纹到中心的距离可求焦距。 解:设第3级暗纹在?3方向上,则有 asin?3?3? 此暗纹到中心的距离为

x3?ftan?3 因为?3很小,可认为tan?3?sin?3,所以

x3?3f?/a 两侧第三级暗纹的距离为

2x3?6f?/a?3mm 所以

f??2x3?a6??25cm

11.18 一单色平行光垂直入射一单缝,其衍射第三级明纹位置恰与波长为600nm的单色光垂直入射该缝时,衍射的第二级明纹位置重合,试求该单色光波长。

分析:所求单色光的第三级明纹位置与波长为600nm单色光的第二级明纹位置重合,说明它们具有相同的衍射角。

?解:单缝衍射明纹位置由asin???2k?1?确定,所以有

2对于波长未知的光?1,

asin?3??2?3?1?对于波长为?2=600nm的光,

asin?2??2?2?1?由于 sin?3?sin?2 由(1)、(2)式可得,

?12 (1)

?22 (2)

?2?3?1??1?2?2?1??22?2

55?600 ?1??2??428.6nm

7711.19 波长为600nm的单色光垂直入射在一光栅上,第二级明纹出现在

sin?=0.2处,第四级缺级。试问:

(1)光栅常数a+b=?

(2)光栅上狭缝的最小宽度a =?

(3)按上述选的(a+b)和a,求出在屏幕上实际呈现的全部衍射明纹的级次。 分析:应用光栅公式和缺级条件可得。 解:(1)由光栅方程

?a?b?sin??k? 得

2?600?10?9k???6?10?6m ?a?b??sin?0.20(2)由缺级条件 k?且第四级缺级,得 a?a?b'a?b'k?k k4a?b'k a当k'=1时,a有最小值 amin?1?a?b??1?6?10?6?1.5?10?6m 44(3)当sin??1时,k有最大值 k?a?b6?10?6??10 ?9600?10?因为当??90?时是看不到衍射条纹的,且k??4,?8时缺级。 能看到的明纹级数为

k?0,?1,?2,?3,?5,?6,?7,?9

11.20 用一束具有两种波长的平行光垂直入射在光栅上,?1?600nm,

?2?400nm,现距中央明纹5cm处?1光的k级明纹和?2光的第k+1级明纹相重合,

若所用透镜的焦距f=50cm,试问:

(1)上述的k=? (2)光栅常数a+b =?

分析:?1光的k级明纹和?2光的第k+1级明纹相重合,即它们的衍射角相同。

解:(1) 由题意,?1的k级与?2的(k+1)级谱线相重合,即它们衍射角相同 所以

dsin?1?k?1,dsin?2??k?1??2,

k??2?1??2?2

(2) 因xf很小, tan?1?sin?1?xf 所以 d?k?1f/x=1.2 ×10?3cm

11.28 两偏振片的偏振化方向成30°夹角时,透射光的强度为I1,若入射光不变而使两偏振片的偏振化方向之间的夹角变为45°,则透光强度将如何变化?

分析:由马吕斯定律求得。 解:由马吕斯定律,

两偏振片的偏振化方向成30°夹角时,

I1?I0cos2?1 (1) 两偏振片的偏振化方向之间的夹角变为45°,

I2?I0cos2?2 (2) 由(1)、(2)得

cos2?2cos245?2 I2???I1

cos2?1cos230?311.29 一束自然光入射到一组偏振片上,该偏振片组共由四块偏振片组成,每块偏振片的偏振化方向均相对于前一块偏振片顺时针转过30°角,问该组偏振片的透射光强占入射光强的百分之几?

分析:由马吕斯定律求得。

解:设入射光的光强为I0,透过第一,二三四块偏振片后的光强分别为

I1,I2,I3,I4。因自然光可以看作振动方向互相垂直,互相独立,光强相等的两个

线偏振光,由第一块偏振片的起偏作用,只透过沿偏振化方向振动的光,所以有

I1?1I0 2射入第二块偏振片的光为线偏振光,透过它的光强为I2,根据马吕斯定律为

I2?I1cos2??同理可得I3和I4

I3?I2cos2??1I0cos4? 261I0cos2? 211?3???0.21I0 I4?I3cos2??I0cos6??I0??22?2??即入射光的21℅能透过偏振片组。

11.32 水的折射率为1.33,玻璃的折射率为1.50。当光由水中射向玻璃而反射时,布儒斯特角为多少? 当光由玻璃射向水中而反射时,布儒斯特角又为多少?

分析:应用布儒斯特定律求得。 解:n1?1.33,n2?1.50 光由水中射向玻璃反射时,

tan?1?n2n1.50?48?26' ,?1?arctan2?arctann1n11.33光由玻璃射向水中反射时,

tan?2?n1n1.33?41?34' ,?2?arctan1?arctann2n21.5011.33 一束太阳光,以某一入射角入射到平面玻璃上,这时反射光为完全偏振光,透射光的折射角为32°。问:

(1)太阳光的入射角是多少? (2)玻璃的折射率是多少?

分析:应用布儒斯特定律求得。

解:(1)反射光为完全偏振光,此时反射光与折射光垂直。 即

i0??0?所以

i0??2

?2??0?90??32??58?

(2)由布儒斯特定律,

tani0?得

n2?tani0?tan58??1.60

加题

加11.1:以单色光照射到相距为0.2mm的双缝上,缝距屏为1m。(1)从第一级明纹到同侧第四级的明纹

为7.5mm时,求入射光波长;(2)若入射光波长为

?n2 n16000A,求相邻明纹间距离。

x??k解:(1)明纹坐标为

由题意有:

D?d,

D?D?3D???dddd0.2?10?3???(x4?x1)??7.5?10?33D3?1x4?x1?4?5?10m?5000A(2)当

?7?。

??6000A时,相邻明纹间距为

?D?1?6000?10?10?3?x???3?10m?3mm?3d0.2?10

加11.2:用?=600 nm的单色光垂直照射在宽为3cm,共有5000条缝的光栅上。问:

(1) 光栅常数是多少?

(2) 第二级主极大的衍射角?为多少?

(3) 若a?2?10(m)光屏上可以看到的条纹的最大级数?

?63.0?10?2?6?10?6(m) 解:(1)光栅常数 d?5000 (2)由光栅方程dsin???k? (k?0,1,2,得

600?10?9sin?2?2?2??0.2 ??2?arcsin0.2?11.5o ?6d6?10?600?10?9??k?0.1 (3) sin?k??k??k??6d6?10??1?sin?k?1

sin?k?10,取k??9, 0.1d考虑缺级k?k3?k?

a?10?k?3,6,9缺级

屏上可以看见的条纹最大级数是8。

加11-3:波长为5000?的单色光正入射到一块缝数为N=10的透射式平面光栅上,两个相邻

5

主极大分别出现在sinθ1=0.15和sinθ2=0.3的位置,并且第四级主极大缺级,求: (1)光栅常数d?(3分)

(2)光栅上每个单缝可能的最小宽度am?(3分) (3)观察屏幕上能哪些谱线?(4分)

解:(1)设相邻主极大分别为m级和m+1级,则:

dsin?1?m?dsin?2?(m?1)?得,d=3.3um

(2)第四级主极大缺级,则kd?4,取k=1,得,每个单缝可能的最小宽度am=0.825um a(3)由dsin??m?得,当sinθ=1时,对应的衍射条纹级数为屏幕上可看到的最大级数,因此,m=6.6,

取整数,屏幕上可以看到的干涉条纹为:0,±1,±2,±3,±4,±5,±6级, 但是第四级主极大缺级,因而最后看到的条纹级数为0,±1,±2,±3,±5,±6级,共

联系客服:779662525#qq.com(#替换为@)